Cardinality of infinite dimensional vector space












0












$begingroup$


Assume that V is an infinite dimensional vector space. I know that if V is a vector space over a field F, then |V|=max{dimV,|F|}. So if we take V=$mathbb{R}$ and F=$mathbb{Q}$ then |V|>|F| and |V|=dimV (Cardinality of a basis of an infinite-dimensional vector space).
Is there any example for the case |V|>dimV and |V|=|F|?










share|cite|improve this question











$endgroup$












  • $begingroup$
    $dim V=1{{}}$?
    $endgroup$
    – Lord Shark the Unknown
    Jan 22 at 20:27










  • $begingroup$
    @LordSharktheUnknown The title says infinite dimensional vector spaces. I don't know if it applies to the desired example. We will see. I would suggest $mathbb R^mathbb N$ over $mathbb R$.
    $endgroup$
    – Dog_69
    Jan 22 at 22:16












  • $begingroup$
    How that works?
    $endgroup$
    – uio666
    Jan 23 at 6:12
















0












$begingroup$


Assume that V is an infinite dimensional vector space. I know that if V is a vector space over a field F, then |V|=max{dimV,|F|}. So if we take V=$mathbb{R}$ and F=$mathbb{Q}$ then |V|>|F| and |V|=dimV (Cardinality of a basis of an infinite-dimensional vector space).
Is there any example for the case |V|>dimV and |V|=|F|?










share|cite|improve this question











$endgroup$












  • $begingroup$
    $dim V=1{{}}$?
    $endgroup$
    – Lord Shark the Unknown
    Jan 22 at 20:27










  • $begingroup$
    @LordSharktheUnknown The title says infinite dimensional vector spaces. I don't know if it applies to the desired example. We will see. I would suggest $mathbb R^mathbb N$ over $mathbb R$.
    $endgroup$
    – Dog_69
    Jan 22 at 22:16












  • $begingroup$
    How that works?
    $endgroup$
    – uio666
    Jan 23 at 6:12














0












0








0


1



$begingroup$


Assume that V is an infinite dimensional vector space. I know that if V is a vector space over a field F, then |V|=max{dimV,|F|}. So if we take V=$mathbb{R}$ and F=$mathbb{Q}$ then |V|>|F| and |V|=dimV (Cardinality of a basis of an infinite-dimensional vector space).
Is there any example for the case |V|>dimV and |V|=|F|?










share|cite|improve this question











$endgroup$




Assume that V is an infinite dimensional vector space. I know that if V is a vector space over a field F, then |V|=max{dimV,|F|}. So if we take V=$mathbb{R}$ and F=$mathbb{Q}$ then |V|>|F| and |V|=dimV (Cardinality of a basis of an infinite-dimensional vector space).
Is there any example for the case |V|>dimV and |V|=|F|?







linear-algebra cardinals






share|cite|improve this question















share|cite|improve this question













share|cite|improve this question




share|cite|improve this question








edited Jan 23 at 6:11







uio666

















asked Jan 22 at 20:22









uio666uio666

62




62












  • $begingroup$
    $dim V=1{{}}$?
    $endgroup$
    – Lord Shark the Unknown
    Jan 22 at 20:27










  • $begingroup$
    @LordSharktheUnknown The title says infinite dimensional vector spaces. I don't know if it applies to the desired example. We will see. I would suggest $mathbb R^mathbb N$ over $mathbb R$.
    $endgroup$
    – Dog_69
    Jan 22 at 22:16












  • $begingroup$
    How that works?
    $endgroup$
    – uio666
    Jan 23 at 6:12


















  • $begingroup$
    $dim V=1{{}}$?
    $endgroup$
    – Lord Shark the Unknown
    Jan 22 at 20:27










  • $begingroup$
    @LordSharktheUnknown The title says infinite dimensional vector spaces. I don't know if it applies to the desired example. We will see. I would suggest $mathbb R^mathbb N$ over $mathbb R$.
    $endgroup$
    – Dog_69
    Jan 22 at 22:16












  • $begingroup$
    How that works?
    $endgroup$
    – uio666
    Jan 23 at 6:12
















$begingroup$
$dim V=1{{}}$?
$endgroup$
– Lord Shark the Unknown
Jan 22 at 20:27




$begingroup$
$dim V=1{{}}$?
$endgroup$
– Lord Shark the Unknown
Jan 22 at 20:27












$begingroup$
@LordSharktheUnknown The title says infinite dimensional vector spaces. I don't know if it applies to the desired example. We will see. I would suggest $mathbb R^mathbb N$ over $mathbb R$.
$endgroup$
– Dog_69
Jan 22 at 22:16






$begingroup$
@LordSharktheUnknown The title says infinite dimensional vector spaces. I don't know if it applies to the desired example. We will see. I would suggest $mathbb R^mathbb N$ over $mathbb R$.
$endgroup$
– Dog_69
Jan 22 at 22:16














$begingroup$
How that works?
$endgroup$
– uio666
Jan 23 at 6:12




$begingroup$
How that works?
$endgroup$
– uio666
Jan 23 at 6:12










0






active

oldest

votes











Your Answer





StackExchange.ifUsing("editor", function () {
return StackExchange.using("mathjaxEditing", function () {
StackExchange.MarkdownEditor.creationCallbacks.add(function (editor, postfix) {
StackExchange.mathjaxEditing.prepareWmdForMathJax(editor, postfix, [["$", "$"], ["\\(","\\)"]]);
});
});
}, "mathjax-editing");

StackExchange.ready(function() {
var channelOptions = {
tags: "".split(" "),
id: "69"
};
initTagRenderer("".split(" "), "".split(" "), channelOptions);

StackExchange.using("externalEditor", function() {
// Have to fire editor after snippets, if snippets enabled
if (StackExchange.settings.snippets.snippetsEnabled) {
StackExchange.using("snippets", function() {
createEditor();
});
}
else {
createEditor();
}
});

function createEditor() {
StackExchange.prepareEditor({
heartbeatType: 'answer',
autoActivateHeartbeat: false,
convertImagesToLinks: true,
noModals: true,
showLowRepImageUploadWarning: true,
reputationToPostImages: 10,
bindNavPrevention: true,
postfix: "",
imageUploader: {
brandingHtml: "Powered by u003ca class="icon-imgur-white" href="https://imgur.com/"u003eu003c/au003e",
contentPolicyHtml: "User contributions licensed under u003ca href="https://creativecommons.org/licenses/by-sa/3.0/"u003ecc by-sa 3.0 with attribution requiredu003c/au003e u003ca href="https://stackoverflow.com/legal/content-policy"u003e(content policy)u003c/au003e",
allowUrls: true
},
noCode: true, onDemand: true,
discardSelector: ".discard-answer"
,immediatelyShowMarkdownHelp:true
});


}
});














draft saved

draft discarded


















StackExchange.ready(
function () {
StackExchange.openid.initPostLogin('.new-post-login', 'https%3a%2f%2fmath.stackexchange.com%2fquestions%2f3083653%2fcardinality-of-infinite-dimensional-vector-space%23new-answer', 'question_page');
}
);

Post as a guest















Required, but never shown

























0






active

oldest

votes








0






active

oldest

votes









active

oldest

votes






active

oldest

votes
















draft saved

draft discarded




















































Thanks for contributing an answer to Mathematics Stack Exchange!


  • Please be sure to answer the question. Provide details and share your research!

But avoid



  • Asking for help, clarification, or responding to other answers.

  • Making statements based on opinion; back them up with references or personal experience.


Use MathJax to format equations. MathJax reference.


To learn more, see our tips on writing great answers.




draft saved


draft discarded














StackExchange.ready(
function () {
StackExchange.openid.initPostLogin('.new-post-login', 'https%3a%2f%2fmath.stackexchange.com%2fquestions%2f3083653%2fcardinality-of-infinite-dimensional-vector-space%23new-answer', 'question_page');
}
);

Post as a guest















Required, but never shown





















































Required, but never shown














Required, but never shown












Required, but never shown







Required, but never shown

































Required, but never shown














Required, but never shown












Required, but never shown







Required, but never shown







Popular posts from this blog

Can a sorcerer learn a 5th-level spell early by creating spell slots using the Font of Magic feature?

Does disintegrating a polymorphed enemy still kill it after the 2018 errata?

A Topological Invariant for $pi_3(U(n))$